Search found 87 matches


the best way to answer the poverty difference and the avg per capita income diff is by choosing option E

In Bahlton, there are people who are too rich and people who are too poor.

so, this answers why poverty as well as avg PCI are greater in Bahlton when compared to Kuptala.

E

by mparakala

Tue Jan 29, 2013 8:02 pm
Forum: Critical Reasoning
Topic: Statistics CR Flaw
Replies: 9
Views: 2639

U----> S (movement of Arbican population) 1980: 13% 1990: 3% number of people moving / total number of people this percentage has to decrease. in option D, if the number of people (numerator) is constant and the total no. of people is decreasing (denominator), then the percentage will increase- this...

by mparakala

Tue Jan 29, 2013 7:46 pm
Forum: Critical Reasoning
Topic: Percentage Critical reasoning from KAPLAN: MEDIUM HARD
Replies: 8
Views: 2952

Thank you for the appreciation David!

by mparakala

Tue Jan 29, 2013 6:52 pm
Forum: Critical Reasoning
Topic: Support--Borania
Replies: 4
Views: 3478

Spanish Armada (modifier prob)

Stymieing the Armada’s plans to meet up with the Duke of Parma’s army off the coast of Flanders in the Spanish Netherlands, the reason for the defeat of the Spanish Armada was not only due to gale winds that favored the British but also the sacrificing of eight war ships as “fireships,� ves...

by mparakala

Tue Jan 29, 2013 2:17 pm
Forum: Sentence Correction
Topic: Spanish Armada (modifier prob)
Replies: 11
Views: 2317

Hi, I clarified my misunderstanding by going back to what I read earlier in the MGMAT guide. Now, it is crystal clear! Thanks for your help :) I don't think I understand your query very well. y has two possible values from the second statement: -8 and 14. When you are combining the two statements yo...

by mparakala

Tue Jan 29, 2013 2:05 pm
Forum: Data Sufficiency
Topic: ABSOLUTE VALUES (CONFUSION!!)
Replies: 4
Views: 1583

Hi, Thank you for responding to my question. Good explanation! Can you give me an example where one of the two values of y is used to obtain an answer? ( as i mentioned earlier, I read it somewhere). For example, in the option (2) , y has 2 values. Is there a situation where one of the two values is...

by mparakala

Tue Jan 29, 2013 10:04 am
Forum: Data Sufficiency
Topic: ABSOLUTE VALUES (CONFUSION!!)
Replies: 4
Views: 1583

ABSOLUTE VALUES (CONFUSION!!)

What is the value of y? (1) 3|x^2 – 4| = y – 2 (2) |3 – y| = 11 Answer: C (source: MGMAT) I learnt somewhere that whenever there is an absolute value question, one obtains two values. The two values must be plugged into the equation normally and checked as one of them will yield a value equiva...

by mparakala

Tue Jan 29, 2013 8:58 am
Forum: Data Sufficiency
Topic: ABSOLUTE VALUES (CONFUSION!!)
Replies: 4
Views: 1583

@brianlange77 , sorry, i forgot to highlight the first sentence which is the boldface statement.

@Mitch, thanks a lot for resolving the confusion :)

by mparakala

Tue Jan 29, 2013 8:48 am
Forum: Critical Reasoning
Topic: Lawyer Reasoning (source:veritas)
Replies: 3
Views: 1521

Lawyer Reasoning (source:veritas)

Lawyer: Many people blame lawyers alone for the decline in people's ability to resolve conflicts outside of the court system. Yet, clearly, plaintiffs' greed has also played an important role in this decline. In the past ten years, lawsuits involving damages of more than one million dollars have ris...

by mparakala

Mon Jan 28, 2013 6:30 am
Forum: Critical Reasoning
Topic: Lawyer Reasoning (source:veritas)
Replies: 3
Views: 1521

Hi, Identifying the conclusion in the stimulus is the best way to start. Conclusion: Non-prescription patches will boost sales look for option that supports this. A: talks about the patches being helpful. That's not what we are looking for B: Nic skin patches cost more than others- then, why buy the...

by mparakala

Sun Jan 27, 2013 10:06 am
Forum: Critical Reasoning
Topic: Support--Borania
Replies: 4
Views: 3478

getting selected for Progaine (the 1st time) = 1/14

(or)

NOT getting selected for Progaine AND getting selected for Ropecia (the 2nd time) = 13/14 * 1/13 = 1/14

1/14 + 1/14 = 2/14 = 1/7

by mparakala

Sun Jan 27, 2013 9:47 am
Forum: Problem Solving
Topic: Ropecia
Replies: 7
Views: 1513

The appropriate conclusion must cover the main points stated in the stimulus. each option highlights a minor point . But C states the entire message of the stimulus- better weapons over the "prior" interest in manpower (earlir, manpower was considered superior but that has changed now). An...

by mparakala

Sun Jan 27, 2013 8:49 am
Forum: Critical Reasoning
Topic: BTG practice questions: military
Replies: 18
Views: 11644

let the principal be "P" 1st year's interest: I = PTR/100 = 6P/100 1st year's amount = P+I = P + 6P/100 = 106P/100 = 1.06P now, 6800 is paid! => 1.06P-6800 is remaining 2nd year's interest = PTR/100 = (1.06P-6800) * 1 * 5 / 100 But, given that this interest is equal to 11/20 of 1st year's ...

by mparakala

Sun Jan 27, 2013 6:10 am
Forum: Problem Solving
Topic: Installment problem
Replies: 2
Views: 1735

. AB + CD AAA if we separately evaluate the left half, A+C must give AA Logically, take the highest single digit number for ex: 9 =>9+9 = 18 that means, when two sngle numbers are added, the sum can be 18 or less than 18. so, out of the many possibilities of AA i.e., 11, 22,33 etc, AA can only take ...

by mparakala

Fri Jan 18, 2013 10:43 am
Forum: Problem Solving
Topic: digit issue
Replies: 26
Views: 13267

X causes Y is the conclusion

option specifying anything other than X causing Y, - weakens the argument

C does the job

teachers methods are not causing decreased numbers.
instead disorder is causing the decreased numbers,

OA: C

by mparakala

Wed Jan 16, 2013 5:25 pm
Forum: Critical Reasoning
Topic: The number of students assigned to special “disruptive
Replies: 6
Views: 2285